1962 AHSME Problems/Problem 14

Revision as of 20:55, 16 April 2014 by Hukilau17 (talk | contribs)

Problem

Let $s$ be the limiting sum of the geometric series $4- \frac83 + \frac{16}{9} - \dots$, as the number of terms increases without bound. Then $s$ equals:

$\textbf{(A)}\ \text{a number between 0 and 1}\qquad\textbf{(B)}\ 2.4\qquad\textbf{(C)}\ 2.5\qquad\textbf{(D)}\ 3.6\qquad\textbf{(E)}\ 12$

Solution

The infinite sum of a geometric series with first term $a$ and common ratio $r$ ($-1<r<1$) is $\frac{a}{1-r}$. Now, in this geometric series, $a=4$, and $r=-\frac23$. Plugging these into the formula, we get $\frac4{1-(-\frac23)}$, which simplifies to $\frac{12}5$, or $\boxed{2.4\textbf{ (B)}}$.